Menu Close

Question-160221




Question Number 160221 by HongKing last updated on 26/Nov/21
Commented by mr W last updated on 26/Nov/21
if question is right, then answer is  wrong. if answer is right, then   question is wrong.  ⇒typo!
$${if}\:{question}\:{is}\:{right},\:{then}\:{answer}\:{is} \\ $$$${wrong}.\:{if}\:{answer}\:{is}\:{right},\:{then}\: \\ $$$${question}\:{is}\:{wrong}. \\ $$$$\Rightarrow{typo}! \\ $$
Commented by quvonnn last updated on 26/Nov/21
$$ \\ $$
Commented by Rasheed.Sindhi last updated on 26/Nov/21
I think sir, tg(87°) instead of tg(78°)
$$\mathrm{I}\:\mathrm{think}\:\mathrm{sir},\:\mathrm{tg}\left(\mathrm{87}°\right)\:\mathrm{instead}\:\mathrm{of}\:\mathrm{tg}\left(\mathrm{78}°\right) \\ $$
Commented by quvonnn last updated on 26/Nov/21
Answered by Rasheed.Sindhi last updated on 26/Nov/21
tg(1).tg(2).tg(3).....tg(87).tg(88).tg(89)=1  tanθtan(90−θ)=1   tan(90−θ)=tan(45+(45−θ))  =((tan45+tan(45−θ)  )/(1−tan45tan(45−θ)  ))   tan(45−θ)=((tan45−tanθ  )/(1+tan45tanθ  ))=((1−tanθ )/(1+tanθ ))  =((1+((1−tanθ )/(1+tanθ ))  )/(1−(1)(((1−tanθ )/(1+tanθ )))  ))=((1+tanθ+1−tanθ  )/(1+tanθ−1+tanθ ))  =(2/(2tanθ ))=(1/(tanθ ))  ∴    tanθtan(90−θ)=1    LHS:  (tan1tan89)(tan2tan88)...(tan45tan45)  =1.1.1....1_(45 times) =1^(45) =1=RHS
$$\mathrm{tg}\left(\mathrm{1}\right).\mathrm{tg}\left(\mathrm{2}\right).\mathrm{tg}\left(\mathrm{3}\right)…..\mathrm{tg}\left(\mathrm{87}\right).\mathrm{tg}\left(\mathrm{88}\right).\mathrm{tg}\left(\mathrm{89}\right)=\mathrm{1} \\ $$$$\mathrm{tan}\theta\mathrm{tan}\left(\mathrm{90}−\theta\right)=\mathrm{1} \\ $$$$\:\mathrm{tan}\left(\mathrm{90}−\theta\right)=\mathrm{tan}\left(\mathrm{45}+\left(\mathrm{45}−\theta\right)\right) \\ $$$$=\frac{\mathrm{tan45}+\mathrm{tan}\left(\mathrm{45}−\theta\right)\:\:}{\mathrm{1}−\mathrm{tan45tan}\left(\mathrm{45}−\theta\right)\:\:}\: \\ $$$$\mathrm{tan}\left(\mathrm{45}−\theta\right)=\frac{\mathrm{tan45}−\mathrm{tan}\theta\:\:}{\mathrm{1}+\mathrm{tan45tan}\theta\:\:}=\frac{\mathrm{1}−\mathrm{tan}\theta\:}{\mathrm{1}+\mathrm{tan}\theta\:} \\ $$$$=\frac{\mathrm{1}+\frac{\mathrm{1}−\mathrm{tan}\theta\:}{\mathrm{1}+\mathrm{tan}\theta\:}\:\:}{\mathrm{1}−\left(\mathrm{1}\right)\left(\frac{\mathrm{1}−\mathrm{tan}\theta\:}{\mathrm{1}+\mathrm{tan}\theta\:}\right)\:\:}=\frac{\mathrm{1}+\mathrm{tan}\theta+\mathrm{1}−\mathrm{tan}\theta\:\:}{\mathrm{1}+\mathrm{tan}\theta−\mathrm{1}+\mathrm{tan}\theta\:} \\ $$$$=\frac{\mathrm{2}}{\mathrm{2tan}\theta\:}=\frac{\mathrm{1}}{\mathrm{tan}\theta\:} \\ $$$$\therefore\:\:\:\:\mathrm{tan}\theta\mathrm{tan}\left(\mathrm{90}−\theta\right)=\mathrm{1} \\ $$$$ \\ $$$$\mathrm{LHS}: \\ $$$$\left(\mathrm{tan1tan89}\right)\left(\mathrm{tan2tan88}\right)…\left(\mathrm{tan45tan45}\right) \\ $$$$=\underset{\mathrm{45}\:\mathrm{times}} {\mathrm{1}.\mathrm{1}.\mathrm{1}….\mathrm{1}}=\mathrm{1}^{\mathrm{45}} =\mathrm{1}=\mathrm{RHS}\:\:\:\:\:\:\:\: \\ $$

Leave a Reply

Your email address will not be published. Required fields are marked *